2004年度後期講義「微分幾何学I」

トップページへ戻る

目次

開講のお知らせ 10月5日 10月12日 10月19日 11月2日 11月9日 11月16日 11月30日 12月7日 12月14日 12月21日 1月11日 1月18日

10月5日(火)開講

2005年1月18日が最終講の予定.


10月26日は, 休講.
11月23日は, 祝日.
1月25日は月曜日に振り替え

授業内容

微分幾何学は, 微分可能多様体やその上の構造(リーマン計量など)を主に微分・ 積分を用いて調べる学問である. この講義では, 調和積分論を目標にリーマン幾何の基本的な内容を取り扱う. 昨年度の授業とは, 内容にあまり重複がないようにする予定である.
単位は, レポートにより判定する. 毎週 1 or 2題の演習問題を授業中に出題するので, 二週間ごとに一題以上を解いて中島 or 数学事務室に提出すること. したがって学期末までに (授業回数/2) 回レポートを提出しなければならない. 最終締め切りは2月1日.

参考文献


10月5日にやったこと

§1. Introduction

$\Omega\subset\mathbf R^n$を有界領域とし, $f$, $u_0$が与えられたときに, 次のDirichlet問題を考える. $$\left\{\begin{array}{ccc} & \Delta u = f & \text{ in } \Omega \\ & u = u_0 & \text{ on }\partial\Omega\end{array}\right.$$ これをDirichlet原理(=Hilbertの直接法)で解く.
$W^{1,2}(\Omega)$を, 1階弱微分が$L^{2}(\Omega)$ に属する関数のなすSobolev空間とする. 内積を $$\displaystyle(u,v)_{W^{1,2}(\Omega)} = (u,v)_{L^2(\Omega)} + \sum_{i=1}^n (\frac{\partial u}{\partial x_i}, \frac{\partial v}{\partial x_i})_{L^{2}(\Omega)}$$ で定義する. この内積のもとでHilbert空間になる.

演習問題1.1

  1. $f(x) = |x|$が $W^{1,2}$に属することを示せ.
  2. $f(x) = \left\{ \begin{array}{cc} 1 & x \ge 0 \\ 0 & x < 0\end{array}\right.$が, $W^{1,2}$に属さないことを示せ.
$C^{\infty}_0(\Omega)$の$W^{1,2}(\Omega)$内での閉包を$W^{1,2}_0(\Omega)$で表わす. これは, `境界値が0であるような$W^{1,2}(\Omega)$の元の全体'という気持を表わしている.

演習問題1.2
$\Omega$が$C^\infty$級境界つき多様体のとき (特に$\partial\Omega$は多様体), $L^2(\partial\Omega)$がしかるべく定義されて, `境界値写像' $W^{1,2}(\Omega)\to L^2(\partial\Omega)$ が定義でき, $W^{1,2}_0(\Omega)$は, この写像のkernelとなることを証明せよ.

$f$, $g^i (i=1,\ldots,n)\in L^2(\Omega)$, $u_0\in W^{1,2}(\Omega)$が与えられたとき, $$\displaystyle\left\{\begin{array}{rcl} - \sum_{i=1}^n \int_\Omega \frac{\partial u}{\partial x_i} \frac{\partial \varphi}{\partial x_i} dx & = & \int_\Omega f \varphi - \sum_i g^i \frac{\partial \varphi}{\partial x_i} dx \quad \forall \varphi\in C^\infty_0(\Omega) \\ u - u_0 & \in & W^{1,2}_0(\Omega)\end{array}\right.$$ が成り立つとき, $u$は $$\displaystyle\left\{\begin{array}{rcl} \Delta u &= &f + \sum_i \frac{\partial g^i}{\partial x_i} \text{ in } \Omega \\ u & = & u_0 \text{ on } \partial\Omega\end{array}\right.$$ の弱解であるという. ($g^i$を導入しておくと, $u - u_0$がみたす方程式を考えるときに便利になる.)

Rieszの表現定理を用いて, 次が示される.

定理. 弱解はただ一つ存在する.


10月12日にやったこと

先週の続き

境界条件 $u_0$ を $0$になるように正規化しておくと, Dirichlet問題の弱解を対応させる写像$L^2(\Omega)^{n+1}\ni (f,g^i)\mapsto u\in W^{1,2}_0(\Omega)$は連続である. すなわち, $$|| u||_{W^{1,2}(\Omega)} \le C (||f||_{L^2(\Omega)} + \sum_i || g^i||_{L^2(\Omega)})$$ を満たす定数 $C$ が存在する. 境界条件 $u_0$ が$0$でないときには, 不等式 $$|| u||_{W^{1,2}(\Omega)} \le C (|| u_0 ||_{W^{1,2}(\Omega)} + ||f||_{L^2(\Omega)} + \sum_i || g^i||_{L^2(\Omega)})$$ が成立する.

トーラスの上のDirichlet問題

$X = \mathbf R^n/\mathbf Z^n$を$n$次元トーラスとする. $X$上の関数 $f$が与えられたとき, 次の問題を考える.

$\Delta u = f$を満たす $X$上の関数 $u$ が存在するか?

簡単に分かることとして, $u_1$, $u_2$が解であるとすると, $u_1 - u_2$ は定数である. また, $\int_X f dx = 0$でないと, 部分積分から解は存在しない.

有限次元の線形写像 $A$ における性質 $f\in\mathrm{Im}A \Leftrightarrow f\perp\mathrm{Ker}A^*$を思いだそう. $\Delta : C^\infty(X)\to C^\infty(X)$の$L^2$内積に関する随伴写像は, $\Delta$であるから, $f\in\mathrm{Im}\Delta$となる必要十分条件が, $f \perp \mathrm{Ker}\Delta = \mathbf R$, すなわち $\int_X f dx = 0$であることは, 上の性質が $A = \Delta$についても成り立っていると仮定すると正しい.

$f\in L^2(X)$としたとき, 上の方程式の $u\in W^{1,2}(X)$における弱解の存在は, 次のPoincar\'eの不等式を示すと, 領域の場合のDirichlet問題と同様に解くことができる.

演習問題2.1 (Poincareの不等式)
次を示せ. (ヒント:フーリエ変換)
ある定数 $C$ が存在して, すべての $u\in W^{1,2}(X)$について $$\displaystyle || u - \int_X u dx ||_{L^2} \le C \sum_i || \frac{\partial u}{\partial x_i} ||_{L^2}$$ が成り立つ.

§2. Riemann計量とラプラシアン

$V$を有限次元実ベクトル空間で, 正定値な内積 $(\ ,\ )$が入っているものとする. このとき `自然'であることの数学的な説明はしない. 少なくとも次のようなことは成り立っている.

演習問題2.2
$\{ e_i\}_{i=1,\ldots,n}$ を $V$の正規直交基底とするとき, $ \{ e_{i_1}\wedge \ldots\wedge e_{i_p} \mid i_1 < \ldots < i_p \}$ は $\Lambda^p V$ の基底になる. これが正規直交基底になるように$\Lambda^p V$に内積を定めるというのが, 上の内積の定め方である. これが最初の正規直交基底 $\{ e_i\}_{i=1,\ldots,n}$の取り方によらないことを証明せよ.

内積に加えて, さらに $\mathrm{vol}\in \Lambda^n V$ で $|\mathrm{vol}| = 1$となるものを取る. ($n = \dim V$) このとき

命題
線形作用素 $\ast : \Lambda^p V \to \Lambda^{n-p} V$ であって $$ (\alpha,\beta)\mathrm{vol} = \alpha\wedge\ast\beta$$ が任意の$\alpha$, $\beta\in\Lambda^p V$について成立するものが, ただ一つ存在する.

$\{ e_i\}_{i=1,\ldots,n}$を$V$の正規直交基底で, $e_1\wedge\ldots\wedge e_n = \mathrm{vol}$となるものを取る. このとき, $I = \{ i_1, \ldots, i_p \}$ に対して $e_I = e_{i_1}\wedge \ldots\wedge e_{i_p}$と定義すると, $$ \ast e_I = \mathrm{sgn}\left(\begin{array}{ccc} I & & I^c \\ 1 & \ldots & n\end{array}\right) e_{I^c}$$ で与えられる. ただし, $I^c$は$I$の補集合である.


10月19日にやったこと

性質 向きをかえると $\ast$ は $-\ast$でおきかえられる.

$\ast\ast = (-1)^{p(n-p)}$が成り立つ.

$\ast 1 = \mathrm{vol}$, $\ast\mathrm{vol} = 1$

$n=2$のとき, $\ast e_1 = e_2$, $\ast e_2 = - e_1$, すなわち, $\ast$は正の向きの90度回転である.

$n=3$のとき $\ast e_1\wedge e_2 = e_3$, $\ast e_1 = e_2\wedge e_3$ etc がなりたつ. ベクトル積と関係がある。

$n=4$のとき $\wedge^2 V$上で $\ast\ast = 1$ が成り立つので, $\wedge^2 V = \wedge^+ V \oplus \wedge^- V$と固有値$\pm 1$の空間に分かれる. 向きに適合した正規直交基底を取れば $$\wedge^\pm V = \mathrm{Span}\{ e_1\wedge e_2\pm e_3\wedge e_4, e_1\wedge e_3 \pm e_4\wedge e_2, e_1\wedge e_4\pm e_2\wedge e_3 \}$$ が成り立つ.

$\{ \theta^i \}$ を正規直交しているとは限らない, しかし向きには適合している基底とし, $g^{ij} = (\theta^i, \theta^j)$と定める. $g^{ij}$を並べてできる行列の逆行列 $(g^{ij})^{-1}$を$g$であらわす.

命題
$(\theta^{i_1}\wedge\ldots\wedge\theta^{i_p}, \theta^{j_1}\wedge\ldots\wedge\theta^{j_p})$は, $g^{-1}$の$i_1$, \ldots, $i_p$行, $j_1$, \ldots, $j_p$列の小行列式で与えられる.
特に $(\theta^1\wedge\ldots\wedge\theta^n, \theta^1\wedge\ldots\wedge\theta^n) = \det g^{-1}$ となり, したがって $\mathrm{vol} = \sqrt{\det g}\theta^1\wedge\ldots\wedge\theta^n$となる.

微分形式 $\alpha = \sum_{i_1<\ldots 演習問題3.1
$$\displaystyle\ast\alpha = \frac1{p!(n-p)!} \sum\mathrm{sgn}\left(\begin{array}{cccccc} i_1 & \ldots & i_p & j_1 & \ldots & j_{n-p} \\ 1 & & \ldots& \ldots & & n\end{array}\right)\sqrt{\det g} g^{i_1k_1} \ldots g^{i_pk_p}\alpha_{k_1\ldots k_p}\theta^{j_1}\wedge\ldots\wedge\theta^{j_{n-p}}$$

定義
$C^\infty$級多様体$X$上のRiemann計量とは, 各点$x\in X$に対して接空間 $T_x X$上の正定値な内積 $g_x$が定められており, 座標 $(x_1,\ldots,x_n)$ を取ったときに $$g_{ij}(x) = g_x (\left(\frac{\partial}{\partial x_i}\right)_x, \left(\frac{\partial}{\partial x_j}\right)_x)$$ で$g_{ij}(x)$を定めると, $x\mapsto g_{ij}(x)$が座標近傍で定義された$C^\infty$級関数になっているときをいう.

以後, $g_x$を単に$g$であらわす.


多様体$X$上にはたくさんのRiemann計量が存在する. $T_x X$上の内積から, 今までのようにして $\wedge^p T_x^* X$上の内積が定められる.

さらに $X$ に向きを与えると, 体積要素 $\mathrm{vol}$ が定まる. 局所座標を使って表わせば $$\mathrm{vol} = \sqrt{\det g} dx_1\wedge\ldots\wedge dx_n$$となる. ただし, $g$ は上の $g_{ij}$ を並べてできる行列である.

Hodgeの$\ast$-作用素 $\wedge^p T^*_x X\to \wedge^{n-p} T^*_x X$が定義される.

$\Omega^p(X)$で$X$上の$p$次微分形式の全体とすると, $\ast$は各点ごとに作用させることによって, $\Omega^p(X)\to \Omega^{n-p}(X)$を定義する.

さらに $X$ はコンパクトであるとし, (任意の微分形式は積分できる) $\Omega^p(X)$上に $L^2$内積を $$(\alpha,\beta)_{L^2} = \int_X g(\alpha,\beta) \mathrm{vol} = \int_X \alpha\wedge \ast \beta$$ によって定義する. 正定値な内積である. ただし, 完備ではない. そこで $L^2$空間を定める.

定義
コンパクト多様体$X$上の$L^2$-$p$次微分形式 $\alpha$ とは, 次のようなものである.

各座標近傍 $(U,x_1,\ldots,x_n)$ごとに$\alpha = \frac1{p!} \sum_{i_1,\ldots,i_p} \alpha_{i_1\ldots i_p} dx_{i_1}\wedge\ldots\wedge dx_{i_p}$ が与えられており, 係数$\alpha_{i_1\ldots i_p}$は, $L^2_{\mathrm{loc}}(U)$である.

二つの座標近傍 $(U,x_1,\ldots,x_n)$, $(V, y_1,\ldots,y_n)$の共通部分では, $$\frac1{p!} \sum_{i_1,\ldots,i_p} \alpha_{i_1\ldots i_p} dx_{i_1}\wedge\ldots\wedge dx_{i_p} = \frac1{p!} \sum_{i_1,\ldots,i_p} \alpha_{j_1\ldots j_p} dy_{i_1}\wedge\ldots\wedge dy_{i_p}$$ (正確には座標変換の意味で)成り立っている.

$L^2$-$p$次微分形式の全体を $L^2(\wedge^pT^*X)$で表わす.

演習問題3.2
$X$はコンパクトとする. 次を示せ.

  1. $L^2(\wedge^pT^*X)$は, 上と同様にして定義される $L^2$内積に関して完備なHilbert空間になる.
  2. $\Omega^p(X)$は, $L^2(\wedge^pT^*X)$の中にdenseに入っている.

11月2日にやったこと

以下, $(X,g)$は向きづけられたコンパクトなRiemann多様体とする.

外微分作用素$d :\Omega^p(X)\to \Omega^{p+1}(X)$の形式的随伴作用素$d^* : \Omega^{p+1}(X)\to\Omega^p(X)$を $$ d^* = (-1)^{np+1} \ast d \ast $$ によって定義する. このとき

命題
$$(d\alpha,\beta)_{L^2} = (\alpha, d^*\beta)_{L^2}$$ が$\alpha\in\Omega^p(X)$, $\beta\in\Omega^{p+1}(X)$について成立する.


$p=1$, $\alpha = \sum \alpha_i dx_i$ のとき, $$d^*\alpha = - \frac1{\sqrt{\det g}} \sum \frac{\partial}{\partial x_j}\left( \sqrt{\det g} g^{ij}\alpha_i\right)$$ が成り立つ. 特に, $(X,g) = (\mathbf R,g_{\text{std}})$のときは, $$ d^*\alpha = -\sum \frac{\partial\alpha_i}{\partial x_i}$$ が成り立つ. $d^*\alpha$はしばしば, divergenceと呼ばれる.

Hodgeのラプラシアン$\Delta$を $$\Delta = d d^* + d^* d$$ で定義する.


$f\in C^\infty(X)=\Omega^0(X)$のとき, $$\Delta f = - \frac1{\sqrt{\det g}} \sum \frac{\partial}{\partial x_j}\left( \sqrt{\det g} g^{ij} \frac{\partial f}{\partial x_i}\right)$$が成り立つ. 特に, $(X,g) = (\mathbf R,g_{\text{std}})$のときは, $$\Delta f = - \sum \frac{\partial^2}{\partial x_i^2} f$$ となり, 通常のラプラシアンの符号を替えたものになっている.

命題
$$(\Delta \alpha,\beta)_{L^2} = (d\alpha,d\beta)_{L^2}+(d^*\alpha,d^*\beta)_{L^2} = (\alpha,\Delta\beta)_{L^2}$$ が成り立つ.

定義
$\alpha$が調和形式(harmonic form)であるとは, $\Delta\alpha = 0$であるときをいう. 上の命題により, $d\alpha = d^*\alpha = 0$と同値である.

定義
$\mathbb{H}^p(X)$を$p$次調和微分形式の全体のなす線形空間とする.

調和積分論の主定理
$$\Omega^p(X) = \mathbb{H}^p(X)\oplus \mathrm{Im}\Delta$$ と$L^2$内積に関して直行分解する.

先の命題により, 二つの部分空間が直行していることは明らか. よって上の定理の主眼は, 次の主張にある
偏微分方程式$\Delta u = \alpha$が解けるための必要十分条件は, $\alpha$が$\mathbb{H}^p(X)$と直行することである.


$\Omega^p(X) = \mathbb{H}^p(X)\oplus \mathrm{Im}d \oplus \mathrm{Im}d^*$と$L^2$内積に関して直行分解する.

さてここで, deRhamコホモロジー群 $H^p(X)$ とは, 複対 $$ \cdots \to \Omega^{p-1}(X) \to \Omega^p(X) \to \Omega^{p+1}(X) \to \cdots$$ について $$\mathrm{Ker}(d:\Omega^p(X)\to \Omega^{p+1}(X))/\mathrm{Im}(d:\Omega^{p-1}(X)\to \Omega^{p}(X))$$ であったことを思いだそう. 上の直行分解において, $\mathbb{H}^p(X)\oplus \mathrm{Im}d $が$\mathrm{Ker}d$に他ならないことに注意すると,


包含写像 $\mathbb{H}^p(X)\to\Omega^p(X)$は, 線形空間の同型写像 $$\mathbb{H}^p(X) \to H^p(X)$$ を導く.

この結果は, 各コホモロジー類の中に調和形式がただ一つ存在することを示している. その意味で, 調和形式のことをharmonic representativeという.

また上の結果と微分トポロジーのよく知られた結果により, $\mathbb{H}^p(X)$が有限次元であることが分かる. この事実の解析的な証明をあとで与える.

系 (Poincar\'e双対性)
pairing $H^p(X)\times H^{n-p}(X)\to \mathbf R$を $$ (\alpha, \beta) \mapsto \int_X \alpha\wedge\beta$$ で定義する. このpairingは非退化で, $H^{n-p}(X)$は$H^p(X)$の双対空間と同型になる.

実際, Hodgeのstar operatorは, 同型写像 $\mathbb{H}^p(X)\to \mathbb{H}^{n-p}(X)$を導くことに注意すると, 上のpairingは$L^2$内積 $(\ ,\ )_{L^2}: \mathbb{H}^p(X)\times \mathbb{H}^p(X)\to \mathbf R$と同一視される. よって非退化である.

. $(X,g) = (\mathbf R,g_{\text{std}})$のときに, 計算してみる.

$\alpha = \alpha_{i_1\cdots i_p}dx_{i_1}\wedge\cdots\wedge dx_{i_p} = \alpha_I dx_I$とすると, $$ d\alpha = \sum_{i\notin I} \frac{\partial \alpha_I}{\partial x_i} dx_i\wedge dx_I$$ となる. また $$ d^*\alpha = \sum_{i\in I} \pm \frac{\partial \alpha_I}{\partial x_i} dx_{I\setminus\{ i\}}$$ となる.

偏微分$\frac{\partial \alpha_I}{\partial x_i}$ が, $i\notin I$と$i\in I$によって分かれて $d\alpha$と$d^*\alpha$に現れていることに注意しよう. 特に, $$ g_{\text{std}}(d\alpha,d\alpha) + g_{\text{std}}(d^*\alpha,d^*\alpha) = \sum_{i=1}^n \left|\frac{\partial\alpha_I}{\partial x_i}\right|^2$$ となることに注意しよう.

演習問題4.1
上の計算式の$\pm$をきちんと計算し, $I = \{i_1, \cdots, i_p \}$と小さい順に並んでいるときに, $$ d^*\alpha = \sum_{l=1}^p (-1)^l \frac{\partial \alpha_I}{\partial x_{i_l}} dx_{I\setminus\{ i_l\}}$$ であることをチェックせよ.


11月9日にやったこと

一般のリーマン多様体$(X,g)$について, 座標系のもとで $\alpha = \frac1{p!} \sum \alpha_{i_1\cdots i_p}dx_{i_1}\wedge\cdots\wedge dx_{i_p}$ とあらわしたとき $$\alpha^{i_1\cdots i_p} = \sum g^{i_1j_1}\cdots g^{i_p j_p} \alpha_{j_1\cdots j_p}$$ と定める. たとえば, 以前にやったことから $$g(\alpha,\beta) = \frac1{p!} \sum \alpha_{i_1\cdots i_p} \beta^{i_1\cdots i_p} = \sum_{i_1 < \cdots < i_p} \alpha_{i_1\cdots i_p} \beta^{i_1\cdots i_p}$$ が成り立つ.

演習問題4.1 (つづき)
$$(d^*\alpha)^{i_1\cdots i_{p-1}} = - \sum_{i=1}^n \frac1{\sqrt{\det g}} \frac\partial{\partial x_i}\left(\sqrt{\det g}\alpha^{i i_1\cdots i_{p-1}}\right)$$ を証明せよ.
ヒント : $(\alpha, d\beta)_{L^2} = (d^*\alpha,\beta)_{L^2}$を, 座標近傍系に台をもった$\beta$について使うと計算が楽になる.

観察.
上の式より, 座標系を $g_{ij}$, $g^{ij}$, $\frac{\partial g_{ij}}{\partial x_k}$が有界になるように取っておくと, ある定数 $C$ が存在して, $$ \frac1C \left( \sum |\alpha_I|^2 + |\frac{\partial \alpha_I}{\partial x_i}|^2\right) \le g(\alpha,\alpha) + g(d\alpha,d\alpha) + g(d^*\alpha,d^*\alpha) \le C\left(\sum |\alpha_I|^2 + |\frac{\partial \alpha_I}{\partial x_i}|^2\right)$$が座標近傍系の各点でなりたつ.

再び$(X,g) = (\mathbf R,g_{\text{std}})$のとき, $\alpha = \alpha_I dx_I$について $$\Delta\alpha = - \sum_{i=1}^n \frac{\partial^2 \alpha_I}{\partial x_i^2}$$ となる.

演習問題5.1
一般の$(X,g)$について, $\Delta\alpha$の主要項($\alpha$の二回微分を含む項)が $$- \sum_{i,j} g^{ij} \frac{\partial^2}{\partial x_i\partial x_j} \alpha_I$$ で与えられることを証明せよ.

§3. 証明ステップ1

$\Omega^p(X)$に$(\ ,\ )_{W^{1,2}}$を $$(\alpha,\beta)_{W^{1,2}} = (\alpha,\beta)_{L^2} + (d\alpha,d\beta)_{L^2} + (d^*\alpha,d^*\beta)_{L^2}$$ によって定義する. $W^{1,2}(\Lambda^p T^*X)$を, $\Omega^p(X)$の$(\ ,\ )_{W^{1,2}}$に関する完備化として定義する. 上の観察を使うと, 演習問題3.2と同様に 各座標近傍 $(U,x_1,\ldots,x_n)$ごとに$\alpha = \frac1{p!} \sum_{i_1,\ldots,i_p} \alpha_{i_1\ldots i_p} dx_{i_1}\wedge\ldots\wedge dx_{i_p}$ が与えられており, 係数$\alpha_{i_1\ldots i_p}$は, $W^{1,2}_{\mathrm{loc}}(U)$であるというものと同じになる.

以下, 座標といったときには, $U$の閉包を含むような開集合上で定義されているものとし, 上の$W^{1,2}_{\mathrm{loc}}(U)$が, $W^{1,2}(U)$になっているようなものを考える.
さらに, そのような座標による開被覆に付随した一の分割も与えられているものとする.
するとそのような座標系ごとに, 各係数のソボレフ内積を考えて, それらをすべて足し合わせてできる内積は, 上の内積と同値になる.

$\alpha\in L^2(\Lambda^pT^*X)$に対し, $\beta\in L^1(\Lambda^{p+1}T^*X)$ が $d\alpha = \beta$を弱い意味で満たすとは, $$(\alpha,d^*\varphi)_{L^2} = (\beta, \varphi)_{L^2}$$ が任意の$\varphi\in\Omega^{p+1}(X)$について成り立つときをいう. $d^*\alpha$についても同様.

このとき, $W^{1,2}(\Lambda^pT^*X)$は, $\alpha\in L^2(\Lambda^pT^*X)$であって, $d\alpha$, $d^*\alpha$がともに存在して, $L^2$に属しているものの全体に他ならない.

$\alpha\in L^2(\Lambda^pT^*X)$, $\beta\in L^2(\Lambda^{p+1}T^*X)$, $\gamma\in L^2(\Lambda^{p-1}T^*X)$に対し, $u\in W^{1,2}(\Lambda^pT^*X)$が (線形)偏微分方程式 $$\Delta u = \alpha + d^*\beta + d\gamma$$ の弱解であるとは, 任意の$\varphi\in\Omega^p(X)$について $$(du, d\varphi)_{L^2} + (d^*u, d^*\varphi)_{L^2} = (\alpha,\varphi)_{L^2} + (\beta,d\varphi)_{L^2} + (\gamma,d^*\varphi)_{L^2}$$ が成り立つときをいう.

定理1
$W^{1,2}(\Lambda^p T^*X)\subset L^2(\Lambda^p T^*X)$はコンパクトな埋め込みである. すなわち, $W^{1,2}(\Lambda^p T^*X)$の有界集合は, $L^2(\Lambda^p T^*X)$で収束する部分列をもつ.

簡単だが大切な応用として次をえる.

命題
調和微分形式の空間 $\mathbb{H}^p(X)$は有限次元である.


11月16日にやったこと

定理2
$u\in W^{1,2}(\Lambda^p T^*X)$が$\Delta u = \alpha + d^*\beta + d\gamma$の弱解であるとし, $\alpha$, $\beta$, $\gamma$は$C^\infty$級であるとする. このとき$u$は自動的に$C^\infty$級になり, $\Delta u = \alpha + d^*\beta + d\gamma$を通常の意味で満たす.

$L^2(\Lambda^p T^*X) = \mathbb{H}^p(X)\oplus \mathbb{H}^p(X)^\perp$と$L^2$内積に関して調和形式の空間の直交補空間を取る.

命題(Poincar\'eの不等式)
ある正の定数$C$が存在して, $u\in \mathbb{H}^p(X)^\perp\cap W^{1,2}(\Lambda^p T^* X)$に対して $$ || u ||_{L^2} \le C \left( || du||_{L^2} + || d^* u||_{L^2}\right)$$ が成立する.

この結果は, トーラスの上の関数($p=0$)の場合の演習問題2.1の一般化である. ただし, トーラスの場合の直接的な証明は適用できない.
背理法により, 列 $u_i\in \mathbb{H}^p(X)^\perp\cap W^{1,2}(\Lambda^p T^* X)$ を $ 1 = || u_i ||_{L^2} \ge i \left( || du_i||_{L^2} + || d^* u_i||_{L^2}\right)$ となるように取り, その極限 $u_\infty$が $\mathbb{H}^p(X)^\perp\cap \mathbb{H}^p(X) = 0$ に属して, $|| u_\infty ||_{L^2} = 1$であるとして矛盾を導く. 定理1, 2 を途中で用いる.

調和積分論の主定理$\Omega^p(X) = \mathbb{H}^p(X)\oplus \mathrm{Im}\Delta$の証明

$\mathcal H = \mathbb{H}^p(X)^\perp\cap W^{1,2}(\Lambda^p T^* X)$に内積を $$\ll \alpha, \beta\gg = (d\alpha,d\beta)_{L^2} + (d^*\alpha,d^*\beta)_{L^2}$$ で定義する. Poincar\'eの不等式により, $\mathcal H$はこの内積のもとでHilbert空間になる. このとき $\beta\in\mathbb{H}^p(X)^\perp$が与えられたとき, $\mathcal H$上の連続線形汎関数$F$を$F(\varphi) = (\beta,\varphi)_{L^2}$で定義する. Rieszの表現定理によって $F(\varphi) = \ll u, \varphi\gg$と表わすと, $u$は$\Delta u = \beta$の弱解となる. さらに$\beta$が$C^\infty$級ならば, 定理2により$u$も$C^\infty$級であり, これが主張に他ならない.

定義
$u$が$\Delta$の固有形式であるとは, $\Delta u = \lambda u$が成り立つときをいい, $0$でない$u$に対してこの式が成り立つとき, $\lambda$は固有値であるという. 固有値$\lambda$の固有形式の全体を$\lambda$-固有空間という.

  1. 固有形式は$C^\infty$級である. (定理2による)
  2. $\lambda \ge 0$
  3. 相異なる固有値に対する固有空間は互いに直交する.
  4. $\lambda$-固有空間の次元は有限次元
  5. 固有値の集合は集積点を持たない.

演習問題6.1
上の性質4,5を証明せよ

min-max原理
固有値を $(0 <) \lambda_1 < \lambda_2 < \cdots$ と小さい順に並べる. 上より$\lambda_i\to \infty$ ($i\to \infty$)となる. このとき $$\displaystyle\lambda_i = \inf\left.\left\{ \frac{(\Delta u, u)_{L^2}}{(u,u)_{L^2}} \right| u\in\Omega^p(X), u\perp \mathbb{H}^p(X)\oplus H(\lambda_1)\oplus \cdots\oplus H(\lambda_{i-1}), u\neq 0\right\}$$ ここで, $H(\lambda_j)$は, $\lambda_j$固有空間である.


11月30日にやったこと

主張
各固有空間の$L^2$内積に関する正規直交基底を, 固有値の小さい順にならべて $u_1$, $u_2, \ldots$ とする. このとき $\{ u_i \}$ は $L^2(\Lambda^p)$の完全正規直交基底をなす. すなわち, 任意の $u\in L^2(\Lambda^p)$に対して $\sum_i (u,u_i)_{L^2} u_i$は$L^2$で$u$に収束する.

演習問題7.1
$u\in W^{1,2}(\Lambda^p)$に対して $\sum_i (u,u_i)_{L^2} u_i$が$u$に$W^{1,2}$で収束すること, $|| u ||_{W^{1,2}}^2 = \sum_i (1+\lambda_i) (u, u_i)_{L^2}^2$となることを示せ.

この基底によって, $L^2$を数列の$\ell^2$空間$\{ (a_i) \mid \sum a_i^2 < \infty\}$と同一視すると, ソボレフ空間は, $\{ (a_i) \mid \sum (1+\lambda_i) a_i^2 < \infty\}$ と同一視される.

§4. ソボレフ空間 (定理1の証明)

$\Omega\subset\mathbf R^n$を有界領域とする.

定理1'
$$W^{1,p}_0(\Omega)\subset\left\{\begin{array}{cc} L^q(\Omega) & p < n \quad q < \frac{pn}{n-p}\\ C^0(\overline\Omega) & p > n \end{array}\right.$$ というコンパクトな埋め込みがある.

$n > 2$, $n=2$, $n=1$で場合分けをする必要があるが, 上の結果から定理1の有界領域版であるところの $$W^{1,2}_0(\Omega)\subset L^2(\Omega)$$ がコンパクトな埋め込みである, という結果が導かれる. さらにコンパクトなリーマン多様体$(X,g)$に対して, 座標系 $\{ (U_a,\varphi_a)\}_{a=1}^N$と付随した$1$の分割 $\{ \rho_a\}$を取ることにより, 上の結果から 定理1の主張が従う.

$\rho$を$\mathbf R^n$上の非負$C^\infty$級関数で, $B_1(0)$に台を持ち, $\int \rho(x) dx = 1$をみたすものとする. これを固定する.
$u\in L^1_{\text{loc}}(\Omega)$と, $h > 0$に対して $$ u_h(x) = \frac1{h^n} \int_{\Omega} \rho(\frac{x-y}h) u(y)dy$$ と定義し, $u$のregularizationという.

$u_h$は, $x$が $\text{dist}(x,\partial\Omega) > h$のときに定義され, 定義されるところで$C^\infty$級である.
$u\in L^1(\Omega)$のときは, $\Omega$の外で$0$とおくことによって, $u_h$は$\mathbf R^n$全体で定義される.

演習問題7.2
次を示せ.
(1) $u\in C^0(\Omega)$のとき, $u_h$は$u$に$\Omega'\subset\subset\Omega$上で一様収束する.
(2) $u\in L^p(\Omega)$のとき, $u_h$は$u$に$L^p(\Omega)$で収束する.
(3) $u\in W^{1,p}(\Omega)$のとき, $\frac{\partial}{\partial x_i} (u_h) = \left(\frac{\partial}{\partial x_i} u\right)_h$が成り立つ.


12月7日にやったこと

ソボレフの埋め込み定理
$$W^{1,p}_0(\Omega)\subset\left\{\begin{array}{cc} L^{np/n-p}(\Omega) & p < n \\ C^{0,1-\frac{n}p}(\overline\Omega) & p > n \end{array}\right.$$ という連続な埋め込みがある.

演習問題8.1
弱微分に関する合成関数の次の微分法則を示せ:
$f$は $\mathbf R$ 上の区分的に微分可能な連続関数で, $f'$は有界とする. $u\in W^{1,1}(\Omega)$のとき, $f\circ u$も$W^{1,1}(\Omega)$に属し, $$\frac{\partial}{\partial x_i}(f\circ u) = f'(u)\frac{\partial u}{\partial x_i}$$ を示せ.

あとは定理1'の証明を行った.


12月14日にやったこと

定理1'の証明のやり直し.

ソボレフの埋め込み定理の弱い主張, $W^{1,p}_0(\Omega)\subset C^0(\overline{\Omega})$ $(p > n)$の証明

§5. 弱解のregularity (定理2の証明)

difference quotientを $$\Delta^h_i u(x) = \frac{u(x+he_i) - u(x)}h$$で定義する.

補題
$u\in W^{1,p}(\Omega)$とする. $\Omega'\subset\subset\Omega$ $\text{dist}(\Omega',\partial\Omega) > h$について $$||\Delta^h_i u||_{L^p(\Omega')}\le || D_i u||_{L^p(\Omega)}$$ が成り立つ.

演習問題9.1
これを証明せよ.

補題
$u\in L^p(\Omega)$ ($p>1$)で, ある定数$K$が存在して, 上と同様の$\Omega'$に対して, $$||\Delta^h_i u||_{L^p(\Omega')}\le K$$ が十分小さい$h$について成り立つとする. このとき, $u$は$x_i$方向に弱微分可能で, $$ || D_i u ||_{L^p(\Omega)}\le K$$ が成り立つ.

$a^{ij}, b^i, c^i, d\in L^\infty_{loc}(\Omega)$, $f, g^i\in L^2_{loc}(\Omega)$とし, 次の線形偏微分方程式を考える. $$ L u = D_i\left(a^{ij} D_j u + b^i u\right) + c^i D_i u + d u = f + D_i g^i$$ (ただし, Einsteinの記法を用い, $\sum$は省略した.)
$u\in W^{1,2}(\Omega)$がこの方程式の弱解であるとは, $$ \int_\Omega \left( a^{ij} D_j u + b^i u\right) D_i \varphi - \left( c^i D_i u + du\right) \varphi dx = \int_\Omega g^i D_i \varphi - f \varphi dx$$ が任意の$\varphi\in C^\infty_0(\Omega)$について成り立つときをいう.

$a^{ij} = a^{ji}$とし, さらに
一様楕円性 ある正の定数$\lambda$が存在して $a^{ij}(x)\xi_i\xi_j \ge \lambda |\xi|^2$が成立する.
係数の有界性 ある定数 $K$が存在して $|a^{ij}(x)|, |b^i(x)|, |c^i(x)|, |d(x)| \le K$が成立する.
と仮定する.

演習問題9.2
$f$, $g^i\in L^2(\Omega)$, $\varphi\in W^{1,2}(\Omega)$について, $$\displaystyle\left\{\begin{array}{lcr} L u &= & f + D_i g^i \text{ in } \Omega \\ u - \varphi & \in & W^{1,2}_0(\Omega)\end{array}\right.$$ の弱解が存在するための必要十分条件を求めよ.

定理2'
さらに, $a^{ij}, b^i\in C^{0,1}(\overline\Omega)$で, $g^i = 0$ i.e., $L u = f$であるとする. このとき, $\Omega'\subset\subset\Omega$について $u\in W^{2,2}(\Omega')$となる.


12月21日にやったこと

定理2'の証明

演習問題10.1
有界領域$\Omega$上のLipschitz連続関数が, 弱微分可能であることを証明せよ.

定理
さらに, $a^{ij}, b^i\in C^{k,1}(\overline\Omega)$, $c^i, d\in C^{k-1,1}(\overline{\Omega})$, $f\in W^{k,2}(\Omega)$で, $g^i = 0$ i.e., $L u = f$であるとする. このとき, $\Omega'\subset\subset\Omega$について $u\in W^{k+2,2}(\Omega')$となる. この定理とSobolevの埋め込み定理を合わせて, 調和積分論の定理2の証明が完了した.

§6. Levi-Civita接続と曲率テンソル

$M$ : $C^\infty$級多様体, $X(M)$ : 多様体上のベクトル場の全体

演習問題10.2
(1) $U\subset M$を開集合とし, ベクトル場 $X$, $Y$, $X'$, $Y'$は $X = X'$, $Y = Y'$を$U$上で満たすとする. このとき $\nabla_X Y = \nabla_{X'} Y'$を$U$上で満たすことを証明せよ.
(2) $\nabla_X Y$の点$x$における値は, 授業で説明したような意味で $X$の点$x$に おける値にしか依存しないことを証明せよ.


1月11日にやったこと

演習問題11.1
上で定められた $\nabla_X \alpha$について $$\begin{array}{cc} & \nabla_X (\alpha\wedge\beta) = \nabla_X \alpha \wedge\beta + \alpha\wedge\nabla_X\beta\\ & X \{ g(\alpha,\beta) \} = g(\nabla_X\alpha,\beta) + g(\alpha,\nabla_X \beta)\end{array}$$ を証明せよ.


1月18日にやったこと

演習問題12.1
$\alpha$が$1$-formのとき, $$\Delta\alpha = \nabla^*\nabla \alpha + Ric(\alpha)$$ を証明せよ.


Ricci曲率が正のとき, $H^1(X,\mathbf R) = 0$となる.


このページの数式は, MathJaxを用いて書かれています
nakajima@math.kyoto-u.ac.jp